Difference between revisions of "2012 AMC 8 Problems/Problem 11"

(Solution)
(Solution)
Line 5: Line 5:
  
 
==Solution==
 
==Solution==
Since there must be an unique mode, and <math>6</math> is already repeated twice, <math>x</math> cannot be any of the numbers already listed (3, 4, 5, 7). (If it were, the mode would not be unique.) So <math>x</math> must be <math>6</math>, or a new number.
+
Since there must be a unique mode, and <math>6</math> is already repeated twice, <math>x</math> cannot be any of the numbers already listed (3, 4, 5, 7). (If it were, the mode would not be unique.) So <math>x</math> must be <math>6</math>, or a new number.
  
 
===Solution 1: Guess & Check===
 
===Solution 1: Guess & Check===

Revision as of 14:02, 1 February 2015

Problem

The mean, median, and unique mode of the positive integers 3, 4, 5, 6, 6, 7, and $x$ are all equal. What is the value of $x$?

$\textbf{(A)}\hspace{.05in}5\qquad\textbf{(B)}\hspace{.05in}6\qquad\textbf{(C)}\hspace{.05in}7\qquad\textbf{(D)}\hspace{.05in}11\qquad\textbf{(E)}\hspace{.05in}12$

Solution

Since there must be a unique mode, and $6$ is already repeated twice, $x$ cannot be any of the numbers already listed (3, 4, 5, 7). (If it were, the mode would not be unique.) So $x$ must be $6$, or a new number.

Solution 1: Guess & Check

We can eliminate answer choices ${\textbf{(A)}\ 5}$ and ${\textbf{(C)}\ 7}$, because of the above statement. Now we need to test the remaining answer choices.

Case 1: $x = 6$

Mode: $6$

Median: $6$

Mean: $\frac{37}{7}$

Since the mean does not equal the median or mode, ${\textbf{(B)}\ 6}$ can also be eliminated.

Case 2: $x = 11$

Mode: $6$

Median: $6$

Mean: $6$

We are done with this problem, because we have found when $x = 11$, the condition is satisfied. Therefore, the answer is $\boxed{{\textbf{(D)}\ 11}}$.

Solution 2: Algebra

Notice that the mean of this set of numbers, in terms of $x$, is:

$\frac{3+4+5+6+6+7+x}{7} = \frac{31+x}{7}$

Because we know that the mode must be $6$ (it can't be any of the numbers already listed, as shown above, and no matter what $x$ is, either $6$ or a new number, it will not affect $6$ being the mode), and we know that the mode must equal the mean, we can set the expression for the mean and $6$ equal:

$\frac{31+x}{7} = 6\\ 31+x = 42\\ x = \boxed{{\textbf{(D)}\ 11}}$

See Also

2012 AMC 8 (ProblemsAnswer KeyResources)
Preceded by
Problem 10
Followed by
Problem 12
1 2 3 4 5 6 7 8 9 10 11 12 13 14 15 16 17 18 19 20 21 22 23 24 25
All AJHSME/AMC 8 Problems and Solutions

The problems on this page are copyrighted by the Mathematical Association of America's American Mathematics Competitions. AMC logo.png